LSAT and Law School Admissions Forum

Get expert LSAT preparation and law school admissions advice from PowerScore Test Preparation.

 Administrator
PowerScore Staff
  • PowerScore Staff
  • Posts: 8917
  • Joined: Feb 02, 2011
|
#24823
Complete Question Explanation

Strengthen—PR. The correct answer choice is (C)

In this stimulus the author discusses a planned art robbery, and the fact that those paintings stolen were not chosen solely based on their value. The author concludes that they were stolen to suit someone’s particular tastes.

The question stem requires us to find the principle appealed to:

Answer Choice (A): The author is not concerned with the question of whether an individual or a group directed the robbery, so this cannot be the principle appealed to.

Answer Choice (B): This principle is contrary to the author’s reasoning—the author believes that some logical justification was involved.

Answer Choice (C): This is the correct answer choice. The author bases conclusions purely on the thieves’ selections, which reflects the belief that such robberies are distinguishable from one another.

Answer Choice (D): The stimulus provides no discussion of any thefts committed without a pre-existing plan—this stimulus deals exclusively with one robbery, which appeared to reflect the existence of a plan.

Answer Choice (E): Since the integrity of the remaining collection is neither mentioned nor alluded to, this cannot be the principle to which the author tacitly appeals.
 Pragmatism
  • Posts: 68
  • Joined: Jan 11, 2018
|
#42816
I am actually having an issue with this question. I chose the answer B, which is clearly wrong. However, I will provide my justification, and I would like your input on it.

B: "Any art theft committed at the direction of a single individual results in a pattern of works taken and works left alone that defies rational analysis."

Key points from the stimulus it addresses: "theft was specifically carried out to suit the taste of some individual collector," and the latter part in the bold supports "clearly had not been greatest estimated market value."

The issue I had with answer choice C was, "can sometime distinguish one type of art theft from another." The issue I had with this was, there could be theft that was carried out to suit the taste of an individual collector who didn't care for market value, which is what the stimulus states differentiates this theft from other thefts. However, I don't see the stimulus isolating that only such thefts, as committed in this stimulus, could be carried out to suit an individual collector who solely wanted the stolen pieces to be of great market value. Thus, if that is the case, then saying, "the pattern of works taken and the works left alone can sometimes distinguish one type of art theft from another," cannot at times distinguish one type of art theft from another.

Am I wrong because the word "sometimes" in answer choice C, though true, is vague to encompass the any granularity of the principle presented in the stimulus, and answer choice B not so? Also, does answer B take "defies rational analysis," to mean make no sense at all, whereas the sense of the theft was premeditated, and thus, rationally coherent in its own nature?
 James Finch
PowerScore Staff
  • PowerScore Staff
  • Posts: 943
  • Joined: Sep 06, 2017
|
#42824
Hi Pragmatism,

As a Strengthen-Principle question, we're looking for an answer choice that makes the conclusion more likely to be true, in an abstract form, however slightly. In this case, there is a clear logical gap between the premises that the pieces were carefully selected, but not for highest market value, and the conclusion that the theft was done at the behest of an individual collector for their private collection. For that conclusion to be true, we can prephrase that the motive for an art theft must be discernible by an analysis of the works taken and not taken.

Answer choice (B) doesn't work because it undermines the logic of the stimulus. The pattern of works taken is being rationally analyzed by the stimulus, with an inference that the works taken means that the theft was for an individual collector, so to say that thefts done on behalf of individual collectors defy rational analysis would invalidate the stimulus's conclusion.

Answer choice (C) is correct because, while written broadly, it makes it more likely that the conclusion in the stimulus is true. The only evidence given for the given conclusion is the analysis of works taken, so if an analysis of only works taken and not taken can at least sometimes distinguish the type of art theft, then it is (slightly) more likely that the conclusion is true, in that this could be one of those times. This also matches up with the prephrase above, albeit in a weaker form.

Hope this clears things up!
 Pragmatism
  • Posts: 68
  • Joined: Jan 11, 2018
|
#42833
Thank you so much
 bkinsella05@gmail.com
  • Posts: 2
  • Joined: Sep 21, 2023
|
#103459
I have been trying to slow down in answering questions and pay more careful attention to the structure and type of each question so that I can use the right tools when answering the question. Upon reading the following Forum Post: Identifying Question Type Inconsistencies. Part 2, I am confused about how this question is classified as a Strengthen-PR question and not a Parallel/Must-PR question. I thought that because the question stem indicates: "The argument tacitly appeals to which one of the following principles," then the principle in the stimulus is being used to affect the principle in the answer choices (Parallel/Must-PR), and not the principle in the answer choice being used to affect the principle in the stimulus (Strengthen-PR). Could someone please provide further clarification?
User avatar
 Jeff Wren
PowerScore Staff
  • PowerScore Staff
  • Posts: 389
  • Joined: Oct 19, 2022
|
#103496
Hi bkinsella,

A principle is "a broad rule that specifies what actions or judgements are correct in certain situations."

In principle questions, you generally either get a principle in the stimulus or you get principles in the answer choices, but not both.

In this question, there is no principle that appears in the stimulus. If you look carefully, you will see that none of the sentences presents a broad rule (either in the premises or the conclusion). However, there is a missing principle underlying the reasoning in the argument. This missing piece of the argument is what we are looking for in the correct answer.

The words "tacitly appeals" in the question stem basically mean "assumes" in this context ("tacitly" meaning implied). While we've classified this as a Strengthen Principle question, the correct answer is also an assumption that is necessary for the argument. Either way, you're looking for the principle that best helps the argument.
User avatar
 Brandon K
  • Posts: 1
  • Joined: May 09, 2023
|
#103500
Thanks Jeff. This was very helpful

Get the most out of your LSAT Prep Plus subscription.

Analyze and track your performance with our Testing and Analytics Package.